[seqfan] Where should I submit this?

Peter Luschny peter.luschny at gmail.com
Tue Apr 15 14:29:02 CEST 2014


MG> Where should I submit these two claims? I would like
MG> to have it entered into Wikipedia.

This is definitely the wrong place. Read Wikipedia's
guidelines [1]. But you should discuss such issues there,
not here.

MG> Two persons have said that for a proof of Claim 2,
MG> the von Mangoldt function series, Abels theorem is
MG> required, but GH from MO at Math Overflow gave a
MG> simpler proof.

If this was discussed on MathOverflow (and as you say proved
by GH from MO) this is perhaps all the help you can get.

It is also visible and well-documented on MO. Therefore
there is IMO no need for further pushing things.

MG> I think these are important.

Even more so in this case. Important stuff will assert
itself quickly.

Peter

[1] http://en.wikipedia.org/wiki/Wikipedia:What_Wikipedia_is_not#Wikipedia_is_not_a_publisher_of_original_thought



More information about the SeqFan mailing list